Đến nội dung

tuan101293 nội dung

Có 316 mục bởi tuan101293 (Tìm giới hạn từ 29-04-2020)



Sắp theo                Sắp xếp  

#221012 Đa thức

Đã gửi bởi tuan101293 on 20-11-2009 - 19:43 trong Phương trình - Hệ phương trình - Bất phương trình

Cho đa thức f(x) bậc n có n nghiệm thực .a là một nghiệm bội của f'(x).CMR:f(a)=0



#220924 giup em voi

Đã gửi bởi tuan101293 on 20-11-2009 - 08:56 trong Phương trình, hệ phương trình và bất phương trình

cho đa thức f(x)=$x^{5}$+$x^{2}$+1 có 5 nghiệm là $x_{1}$,$x_{2}$,$x_{3}$,$x_{4}$,$x_{5}$. Đặt Q(x)=$x^{2}$-2.
Tính Q($x_{1}$)Q($x_{2}$)Q($x_{3}$)Q($x_{4}$)Q($x_{5}$).

đáp án là 71



#220921 bất đẳng thức c/m bằng quy nạp nè

Đã gửi bởi tuan101293 on 20-11-2009 - 08:49 trong Bất đẳng thức và cực trị

có bài này ai rảnh trả lời nha ( có trong toán tuoi tho )
so sánh $ 1999^{1998}$ và $ 1998^{1999} $
bít kq nhưng quên mât cách làm

ta có $1998^{1999}>1999^{1998}$
bạn có thể CM như sau
xét hàm $f(x)=\sqrt[x]{x}$
$f'(x)=\sqrt[x]{x}*(\dfrac{1-ln(x)}{x^2})<0$ với x>3
suy ra $\sqrt[1998]{1998}>\sqrt[1999]{1999}$ .....



#220918 Bài cũ

Đã gửi bởi tuan101293 on 20-11-2009 - 08:33 trong Bất đẳng thức - Cực trị

bạn có thể post lời giải dùng p,q,r đc ko?



#220917 1/a^2+2bc

Đã gửi bởi tuan101293 on 20-11-2009 - 08:32 trong Bất đẳng thức - Cực trị

Tớ gõ các này mục đích là tập tành cái chuyện soạn văn bản bằng latex thôi :(

Tớ lần đầu gõ latex nên mò mẫm có hơi lâu :leq ( độ .... 4 -5 tiếng gì đấy :leq ) , lệnh thì tớ biết rất ít ---> trình bày có hơi khó coi 1 xíu mong mọi người bỏ qua cho :luoi .

Cái lời giải này của VIMF tớ cũng chưa check nữa ... anh em xem có thấy cái gì sai thì nói tớ sửa nhá :luoi

Thì mình cũng bảo là 27 thì có lời giải đẹp mà:leq
còn cái 310 thì nhìn kinh quá,ko dám expand.
thks
p/s:lời giải quy đồng của mình đẹp hơn:D



#220903 1 bai toan luong giac

Đã gửi bởi tuan101293 on 19-11-2009 - 22:38 trong Các dạng toán khác

chung minh:
cos72= :frac{3- :sqrt{5} }{2( :sqrt{5} -1} chua biet cach go cong thuc mong moi nguoi thong cam

Bài này có trong sbt lớp 10 mà
CM bằng cách dựng tam giác có 3 góc là 72,72,36....



#220840 1/a^2+2bc

Đã gửi bởi tuan101293 on 19-11-2009 - 16:25 trong Bất đẳng thức - Cực trị

Thay đổi một chút nhá!!!! Hãy cm:
$\dfrac{1}{2a^2+bc}+\dfrac{1}{2b^2+ca}+\dfrac{1}{2c^2+ab} \ge \dfrac{ab+bc+ca}{a^2b^2+b^2c^2+c^2a^2}$
:(

Công nhận là thế thật:
ta có $a^2b^2+b^2c^2+c^2a^2\ge 2bca^2+b^2c^2=bc(2a^2+bc)$
tương tự ghép vào ta có đpcm



#220444 cần tài liệu về số nguyên Gauss,các bài toán dùng số phức để giải pt nghiệm n...

Đã gửi bởi tuan101293 on 15-11-2009 - 18:22 trong Tài nguyên Olympic toán

Mình cần tài liệu về số nguyên Gauss,các bài toán dùng số phức để giải pt nghiệm nguyên
Bạn nào có thì share mình với
cảm ơn nha



#220443 1/a^2+2bc

Đã gửi bởi tuan101293 on 15-11-2009 - 18:15 trong Bất đẳng thức - Cực trị

Với cùng điệu kiện trên thì bất đẳng thức sau vẫn đúng :

$\dfrac{1}{{{a^2} + 2bc}} + \dfrac{1}{{{b^2} + 2ca}} + \dfrac{1}{{{c^2} + 2ab}} \ge \dfrac{{ab + bc + ca}}{{{a^2}{b^2} + {b^2}{c^2} + {c^2}{a^2}}} + \dfrac{{37}}{2}.\dfrac{{{{\left[ {\left( {a - b} \right)\left( {b - c} \right)\left( {c - a} \right)} \right]}^2}}}{{{{\left( {a + b + c} \right)}^6}\left( {ab + bc + ca} \right)}}$

Mình nghĩ là bdt này có vấn đề
chắc là phải thay 37 thành 27 và 1 số cái nữa



#220435 1/a^2+2bc

Đã gửi bởi tuan101293 on 15-11-2009 - 17:25 trong Bất đẳng thức - Cực trị

mình giải qua thế này nhé
Ta xét TH a,b,c>0
chú ý là
$\sum_{cyc}\dfrac{1}{a^2+2bc}=\dfrac{(\sum ab)(2\sum a^2+\sum ab)}{\prod(a^2+2bc)}$
tức là ta sẽ CM
$(2\sum a^2+\sum ab)(\sum a^2b^2)\ge \prod(a^2+2bc)$
(mình định thử côsi nhưng mà phải ăn cơm nên làm vội phân tích kiểu SOS và schur)
expand ra ta phải CM
$\sum_{cyc}2a^4(b-c)^2-\sum a^2b^2(c-b)(c-a)\ge 0$
đúng theo côsi 2 số
ĐPCM



#220425 Đề thi học sinh giỏi toán 12 thành phố hà nội năm 2009-2010

Đã gửi bởi tuan101293 on 15-11-2009 - 16:37 trong Thi HSG cấp Tỉnh, Thành phố. Olympic 30-4. Đề thi và kiểm tra đội tuyển các cấp.

Đề thi học sinh giỏi thành phố hà nội - lớp 12
Năm 2009-2010
BÀI 1
cho hàm số $y=({{x}^{2}-1})^{2}-({m+1})^{2}({1-m})^{2}$
( m là tham số)
1.Biện luận theo m số giao điểm của đồ thị hàm số với trục hoành
2.Xác định m để đồ thị hàm số cắt trục hoành tại 4 điểm phân biệt có hoành độ tương ứng lập thành cấp số công.
BÀI 2
1.giải phương trình $9\left(\sqrt{4x+1}-\sqrt{3x-2} \right)=x+3$
2.Cho dãy số $u_{n}=\dfrac{P_{n}}{{A_{n+2}}^n}$trong đó số chỉnh hợp chập n của (n+2) phần tử là ${A_{n+2}}^n}$ và $P_{n}$ là số hoán vị của tập hợp gồm n phần tử với n là số nguyên dương. Tìm $lim S_{n}= \sum\limits_{i=1}^{n} u_{i} $
BÀI 3
Cho hình lập phương ABCDA'B'C'D' cạnh bằng a .Với M là một điểm thuộc cạnh AB, Chọn điểm N thuộc cạnh D'C' sao cho AM+D'N=a
1.Chứng minh MN đi qua một điểm cố định khi M thay đổi
2.tính thể tích chóp B'.A'MCN theo a. Xác định vị trí của M để khoảng cách từ B' tới mp(A'MCN) max.Tính khoảng cách lớn nhất đó theo a.
3.Tìm quỹ tích hình chiếu vuông góc của C xuống MN khi M chạy trên AB.
BÀI 4
1. Cho hai số thực $x,y$ thỏa mãn $1\geq x\geq y>0$
chứng minh: $\dfrac{{x}^{3}{y}^{2}+{y}^{3}+{x}^{2}}{{x}^{2}+{y}^{2}+1}\geq xy$
2. Viết phương trình của đường thẳng tiếp xúc với đồ thị hàm số $y=(x-1)({x}^{3}+{x}^{2}+1)$ tại hai điểm phân biệt thuộc đồ thị hàm sô.
HẾT
*****************



#220203 Bài bdt trong kỳ thi hsg thành phố HN vòng 1 năm 2009

Đã gửi bởi tuan101293 on 12-11-2009 - 20:22 trong Bất đẳng thức - Cực trị

cho x,y thỏa mãn:
$1\ge x\ge y>0$
CMR:
$\dfrac{x^3y^2+y^3+x^2}{x^2+y^2+1}\ge xy$



#219854 Tìm cực trị

Đã gửi bởi tuan101293 on 08-11-2009 - 17:50 trong Bất đẳng thức và cực trị

Với x như thế nào vậy? dương, âm hay nguyên dương?Nếu x nguyên dương thì ta có thể xét x=0 rồi với x nguyên dương ta có bđt$ \dfrac{a^x+b^x}{2} \geq (\dfrac{a+b}{2})^x$

với x>1 hoặc x<0 ta có $\dfrac{a^x+b^x}{2}\ge (\dfrac{a+b}{2})^x$
vơi 0<x<1 ta có $\dfrac{a^x+b^x}{2}\le (\dfrac{a+b}{2})^x$
với x=0,1 ta có đẳng thức



#219824 TRỨNG CHỌI ĐÁ

Đã gửi bởi tuan101293 on 08-11-2009 - 12:07 trong Phương trình - Hệ phương trình - Bất phương trình

$(a,b,c,d)=(a,1-a,1-a(1-a),1-a(1-a)(1-a(1-a)))$



#219570 Bài lượng giác

Đã gửi bởi tuan101293 on 04-11-2009 - 19:34 trong Các bài toán Đại số khác

Theo mình nếu cos(x)>=0 thi bdt này mới đúng
nếu với mọi x thì thử x=140 độ thì sai



#219446 mọi người sao không ai post bài mới lên à

Đã gửi bởi tuan101293 on 02-11-2009 - 20:05 trong Bất đẳng thức - Cực trị

thí điểm bài này nhé em
a+b+c=1,a,b,c>0
CMR:
$\dfrac{(48abc+1)}{abc}\ge \dfrac{25}{ab+bc+ca}$